Geometrie
Répondre à la discussion
Affichage des résultats 1 à 24 sur 24

Geometrie



  1. #1
    invitec7b3f097

    Geometrie


    ------

    Soit un n-agone convexe A1A2...An. Soit P un point a l'interieur de celui-ci.
    Montrer que l'un des angles PA1A2,PA2A3,....PAnA1 est inferieur ou egal a (n-2)pi/(2n)

    -----

  2. #2
    Rincevent

    Re : Geometrie

    Bonjour,

    ce forum n'est aucunement un self-service pour déposer des énoncés d'exercices. Il n'y est de plus aucunement interdit d'y utiliser des formules du genre "bonjour", "svp", etc. Voire c'est même fortement recommandé par la charte que tu es supposé(e) avoir lue en t'inscrivant:

    les règles

    merci d'essayer de t'adapter à cela à l'avenir...

    Pour la modération,
    Rincevent

  3. #3
    invitec7b3f097

    Re : Geometrie

    Excusez-moi, j'avais pas lu la charte !

    Je reposte le probleme:

    Bonjour,

    Pourriez-vous m'aider dans la resolution de cet exercice?
    Soit un n-agone convexe A1A2...An. Soit P un point a l'interieur de celui-ci.
    Montrer que l'un des angles PA1A2,PA2A3,....PAnA1 est inferieur ou egal a (n-2)pi/(2n)

    Merci.

  4. #4
    Rincevent

    Re : Geometrie

    c'était peut-être pas la peine de redonner l'énoncé...

    simplement indépendamment de la charte, je pense que de toutes façons le fait de dire des trucs comme "bonjour" et "merci" ne peut qu'augmenter tes chances d'obtenir une réponse...

    à part ça, je n'ai pas le temps de regarder ton truc en détails, mais as-tu essayé de faire une démo par l'absurde où tu supposes que tous tes angles sont strictement supérieurs à cette valeur? en faisant ainsi tu trouverais peut-être que la somme de tous les angles ainsi obtenus serait peut-être plus grande qu'une limite à ne pas dépasser (du genre 2 pi).

    m'enfin, je te dis ça sans avoir vraiment réfléchi au problème...

  5. A voir en vidéo sur Futura
  6. #5
    invitec7b3f097

    Re : Geometrie

    Bonjour,

    Oui j'ai essaye cette approche mais ca ne donne rien

    Bonne journee

  7. #6
    Rincevent

    Re : Geometrie

    tu n'es pas obligé de dire "bonjour" à chaque message non plus...

  8. #7
    invite69dafe8b

    Re : Geometrie

    soit un carré! polygone convexe a 4 coté!
    Soit P le centre du carré!
    l'angle A1PA2 = A2PA3 = A3PA4 = A4PA1 = Pi/2
    (car A1P est orthogonal a A2P)

    et ton hypothese dit que l'un des angles devrais etre égal ou inferieur à (n-2)pi/2n
    (4-2)Pi/8 = 2Pi/8 = Pi/4

    donc pour moi, ta proposition est fausse! (A confirmé, car fait en vitesse)

  9. #8
    invitec7b3f097

    Re : Geometrie

    Bonjour,

    Hehe certes, mais ce ne sont pas les angles A1PA2,...., qu'on regarde mais les angles PA1A2,....

    Bonne journee

  10. #9
    shokin

    Talking Re : Geometrie

    salut, Lord

    je vais essayer de démontrer cela :

    soit un polygone à n côtés et un point P au sein de ce polygone.
    je relie le point P à chaque sommet et j'obtiens 5 triangles.

    A chaque triangle, je trace la hauteur allant de B1 à P, B1 étant situé entre les sommets A1 et A2. J'ai donc 10 triangles rectangles :

    je démontre d'abord que tous angles PA1A2, ..., PAnA1 ne peuvent pas tous être plus grands (ni plus petits d'ailleurs) que leurs correspondants PA2A1, ..., PA1An :

    Soient les triangles PA1B1 et PA2B1. Ils ont un côté commun et un angle égal (l'angle droit). De plus B1 est situé sur le segment [A1;A2]. Donc si l'angle PA1B1 (=PA1A2) est plus grand que l'angle PA2B1, alors le segment PA2 est plus grand que le segment PA1.

    Tous les angles PA1A2, ..., PAnA1 ne peuvent donc pas être plus grands que leurs correspondants respectifs PA2A1, ..., PA1An, car dans ce cas alors PA1>PAn>PAn-1>...>PA2>PA1 donc PA1>A1 ce qui est absurde.

    Donc il existe au moins un angle PA1A2, ..., PAnA1 , appelons-les groupe 1, qui soit plus petits que son correspondant respectif PA2A1, ..., PA1An, appelons-les le groupe 2.

    Dans un polygone régulier à n côtés, tous les angles tant du groupe 1 que du groupe 2 mesurent pi(n-2)/2n, la valeur que tu as citée.

    Dans tout polygone à n côtés, la sommes des angles des groupes 1 et 2 ensembles égale pi(n-2). La valeur pi(n-2)/2n est donc la moyenne arithmétique de ces 2n angles. On l'appellera S.

    Tous les angles du premier groupe ne peuvent être plus strictement plus grand que cette valeur moyenne car admettons qu'ils le soient : ils ne sont pas tous plus grands que leurs correspondants respectifs du deuxième groupe, en raison du théorème démontré précédemment.

    Il y en a donc un qui est plus petit que son correspondant (je me répète). S'il est plus grand que S alors son correspondant l'est aussi. Mais dans ce cas, la somme des dix angles excède pi(n-2).

    Donc il est plus petit que S.

    CQFD

    je sais c'est pas très clair, j'aurais bien voulu trouver une démonstration simple, claire, rapide et juste, mais... je vais essayer.

    C'est même possible que je me sois trompé, que ça ne m'étonnerait pas.
    Pardon, humilité, humour, hasard, tolérance, partage, curiosité et diversité => liberté et sérénité.

  11. #10
    invite37968ad1

    Re : Geometrie

    Très jolie démonstration pour prouver qu'il existe au moins un angle plus petit que son correspondant!

    mais la conclusion "Il y en a donc un qui est plus petit que son correspondant (je me répète). S'il est plus grand que S alors son correspondant l'est aussi. Mais dans ce cas, la somme des dix angles excède pi(n-2)" ne me convainct pas: pourquoi ne pourrait-t-il exister un correspondant plus grand que S, tant qu'il y a suffisamment de correspondants plus petits que S ?

  12. #11
    shokin

    Talking Re : Geometrie

    sur la fin, je n'arrive pas à trouver une proposition sans contre-exemple (malheureusement il y en a !) . Je vais alors me lancer sur une autre piste.

    Soit un polygone L à n côtés.
    Soit un point P seyant au sein d'L.

    Je relie tous les points A1, ..., An au point P par des segments.

    Si les angles du groupe 1, càd PA1A2, ..., PAnA1 augmente tous, sans que les sommets A1, ..., An bougent, les segments qui concouraient en P ne sont plus tous concourants en un seul point.

    L'implication contraposée est donc :

    si le point P, qui est unique, se déplace et donc reste unique, avec toujours à sa suites les segments qui le relient aux sommets, alors au moins un des angles du groupe 1 a diminué (et même au moins un a augmenté).

    donc si le point P se déplace, au moins 1 angle du groupe 1 (et même aussi du groupe 2) augmente et au moins 1 diminue.

    Maintenant si un sommet se déplace :

    prenons pour exemple le sommet A2,

    considérons une droite séparant les sommets A1 et A3, par exemple la droite (PA2) (mais n'importe laquelle qui sépare A1 et A3 aurait joué, d'ailleurs nous en prendrons une autre).

    Si le point A2 se déplace sur cette droite, des deux angles du groupe 1 PA1A2 et PA2A3, l'un augmente, l'autre diminue car si les deux augmentaient (ou diminuaient), le point A2 se diviseraient en deux points qui s'éloigneraient, et A2 ne serait plus unique.

    donc si un point se déplace sur une droite qui sépare deux autres points, des deux angles du même groupe qui dépendaient de ce point, l'un augmente, l'autre diminue.

    je peux donc considérer une autre droite qui sépare ces deux mêmes points A1 et A3, mais qui ne soit pas parallèle à la première droite. Si le point A2 se déplace sur cette droite, je suis le même raisonnement, l'un des deux angles PA1A2 et PA2A3 diminue, l'autre augmente.

    Le point A2 peut alors se déplacer en augmantant un des angles cités et en diminuant l'autre, appelons-les son "go" et son "gogo", dans tout le plan hormis sur la droite (A1A3) auquel cas L n'a plus n côtés.

    Donc si P se déplace au sein d'L ou si L veut modifier son allure en faisant bouger ses n sommets, tant qu'elle en garde n. Parmi les anges du groupe 1, respectivement du groupe 2, l'au moins un augmente, l'au moins autre diminue.

    Imaginons qu'L ait la forme la plus parfaite avec ses n côtes, L est un polygone régulier donc les angles du groupe 1 ainsi que ceux du groupe 2 mesurent chacun 2pi(n-2)/2n (étant donné que la somme des angles du milieu (A1PA2, ..., AnPA1) est égale à 2pi et que la somme des angles des n triangles égale n*pi, la somme des angles des groupes 1 et 2 ensembles mesure alors la différence, càd pi(n-2), et comme ils sont tous égaux et comme il y en a 2n, chacun mesure 2pi(n-2)/2n ), la valeur critique que tu as citée, Lord.

    Donc si le point P est capable de se déplacer partout au sein d'L et si les n sommets d'L se meuvent aussi, L peut requérir toutes les allures possibles, c'est donc valable pour tout polygone à n côtés.

    Je peux partir du polygone régulier dont je connais la mesure des angles du groupe 1. Comme à chaque pulsation, l'un des joueurs de go gagne et un autre des joueurs à gogo perd, il y en aura toujours au moins un qui au pire rien perdu et un autre qui au mieux n'aura rien gagné.

    Donc pour tout polygone L dans le plan, il existe au moins un angle parmi les angles PA1A2, ..., PAnA1 qui soit inférieur ou égal à (n-2)pi/(2n), pour répondre à ta question. (on peut bien sûr dire qu'il y en a toujours un qui soit supérieur ou égal à (n-2)pi/(2n), et de même pour les angles du groupe 2).

    CQFD

    J'arrête là pour cette démo. Là je suis sûr de mon raisonnement. Pas comme pour la première démo.
    Pardon, humilité, humour, hasard, tolérance, partage, curiosité et diversité => liberté et sérénité.

  13. #12
    invite37968ad1

    Re : Geometrie

    Citation Envoyé par shokin

    Maintenant si un sommet se déplace :

    prenons pour exemple le sommet A2,

    considérons une droite séparant les sommets A1 et A3, par exemple la droite (PA2) (mais n'importe laquelle qui sépare A1 et A3 aurait joué, d'ailleurs nous en prendrons une autre).

    Si le point A2 se déplace sur cette droite, des deux angles du groupe 1 PA1A2 et PA2A3, l'un augmente, l'autre diminue car si les deux augmentaient (ou diminuaient), le point A2 se diviseraient en deux points qui s'éloigneraient, et A2 ne serait plus unique.
    je suis toujours aussi difficile (je me demande si je ne vais pas changer de pseudo et m'appeler St Thomas...)
    Pour que l'angle PA1A2 augmente, il suffit de déplacer A2 dans le demi-plan de frontière A1A2 ne contenant pas P
    Pour que PA2A3 augmente, il suffit de déplacer A2 à l'intérieur du cercle circonscrit au triangle PA2A3. Le cercle et le demi-plan ont en général une intersection non vide, donc on peut arriver à déplacer A2 en augmentant à la fois PA1A2 et PA2A3 (évidemment pas si on se déplace sur PA2, mais dès qu'on prend une droite quelconque)
    Raisonnement analogue pour diminuer à la fois les deux angles

    oui, je sais...., je ferai mieux de proposer une solution mais comme je n'en trouve pas pour l'instant je me venge!!!

  14. #13
    shokin

    Talking Re : Geometrie

    curieux, viendrais-tu de trouver un contre-exemple au théorème que nous essayons de démontrer ? peut-être, peut-être pas...

    parce que prenons le polygone régulier à n côtés : si l'on considère le demi-plan ainsi que cercle que tu as cités, il y a bien une intersection entre les deux, donc les 2 angles ont augmenté tandis que les autres n'ont pas changé... quel contre-exemple !

    mais je vais encore réfléchir dans quels cas leur intersection n'est qu'un point, auquel cas les deux angles ne peuvent pas augmenter simultanément...

    ou j'irai voir d'un autre côté encore...
    Pardon, humilité, humour, hasard, tolérance, partage, curiosité et diversité => liberté et sérénité.

  15. #14
    shokin

    Talking Re : Geometrie

    oooouuups ! ce que je viens de dire au dernier post n'est pas un contre-exemple, puisque les autres angles sont égaux à pi(n-2)/2n.

    mais d'un certain point de vue, ça paraît évident, c'est donc difficile (mais possible !) de démontrer ce théorème, qui n'est donc pas un axiome.
    Pardon, humilité, humour, hasard, tolérance, partage, curiosité et diversité => liberté et sérénité.

  16. #15
    invite5b168fdb

    Re : Geometrie

    je trouve également un contre-exemple dans le cas d'un triangle équilatéral.
    une analyse rapide du pb (et non vérifié par la preuve) semble permettre de postuler que ce résultat n'est vrai qu'à partir de n=6. dans ce cas, il est facile de démontrer ce résultat par l'absurde, comme proposé par rincevent au début de ce fil

  17. #16
    invite37968ad1

    Re : Geometrie

    j'aimerais bien voir ce contre-exemple!!

    Car, on démontre très simplement que la propriété est vrai pour tout polygône régulier.
    Il suffit de considerer le centre O du polygône régulier. Les segments [OA1], [OA2], ...[OAn] découpent le polygone en n triangles isocèles de sommet O ( angle au sommet 2pi/n ; angle des bases (1/2)(pi - 2pi/n) = (n-2)pi/(2n) )

    Le point P est nécessairement dans l'un de ces triangles OAkA(k+1)](éventuellement au bord), donc l'angle PAkA(k+1) est inférieur ou égal à OAkA(k+1) donc inférieur ou égal à (n-2)pi/(2n)

    Le problème est dans la généralisation à un polygone quelconque.

    Quant à la démonstration par l'absurde "facile" pour n > 6 avec un polygone quelconque, je demande à voir....

  18. #17
    invitec7b3f097

    Arrow Re : Geometrie

    "Je trouve également un contre-exemple dans le cas d'un triangle équilatéral"

    Hehe... Je demande a le voir aussi, surtout que c'etait un probleme 5 d'Olympiade Internationale

  19. #18
    shokin

    Talking Re : Geometrie

    je suis sûr que le théorème est vrai, j'en suis convaincu !

    mais j'ai toujours pas trouvé comment démontrer simplement... et pourtant je suis sûr qu'il doit y avoir une démo assez simple, assez rapide ! j'essaie encore...
    Pardon, humilité, humour, hasard, tolérance, partage, curiosité et diversité => liberté et sérénité.

  20. #19
    invitec7b3f097

    Re : Geometrie

    Voulez-vous que je poste une solution?

  21. #20
    invitec7b3f097

    Re : Geometrie

    Bon bah j'en poste deux:

    Solution de P.Bornsztein:
    Soit Ri=PAi , xi=PAiA(i+1) (l'angle) et yi= PAiA(i-1) (l'angle).
    Les indices sont consideres modulo n.
    On a bien sur Somme (de 1 a n) xi+yi=(n-2).pi .

    La loi des sinus donne: Ri/R(i+1)=sin(x(i-1))/sin(yi).
    On fait le produit de ces relations:
    Produit (de 1 a n) sin(xi)= Produit (de 1 a n) sin (yi). Soit p la valeur commune.
    On en deduit d'apres l'inegalite arithmetico-geometrique, et de la concavite de sin sur [0,pi]:
    p^2=Produit (de 1 a n) sin(xi).sin(yi) <= ((Somme (de 1 an) sin(xi)+sin(yi))/(2n))^(2n)
    <= sin((n-2)pi/(2n))^(2n)
    <=sin((n-2)pi/(2n))^n.
    Il existe donc i tel que sin (xi)<=sin((n-2)pi/(2n))

    Ma solution:
    L'exercice devient trivial si on connait l'inegalite d'Erdos-Mordell generalisee:
    Soit Bi le projete de P sur le cote AiA(i+1), avec les indices consideres modulo n.
    PA1+PA2+---+PAn>= 1/cos(pi/n)(PB1+....+PBn).
    La loi des sinus donne PBi=PAi.sin(PAiA(i+1)) et c'est fini !

  22. #21
    shokin

    Talking Re : Geometrie

    désolé Lord, je ne remets nullement en question tes démonstrations, en fait j'ai pas compris.

    tu pourrais m'expliquer ce que sont :

    la loi des sinus

    l'inégalité arithmético-géométrique

    la concavité de sin sur [0,pi]

    l'inégalité d'Erdos Mordell.

    Parce que, pour moi, qui ne connaît pas ces termes, le problème n'est pas encore trivial. Dieu sait pour ceux qui comme moi...
    Pardon, humilité, humour, hasard, tolérance, partage, curiosité et diversité => liberté et sérénité.

  23. #22
    invitec7b3f097

    Re : Geometrie

    Citation Envoyé par shokin
    désolé Lord, je ne remets nullement en question tes démonstrations, en fait j'ai pas compris.

    tu pourrais m'expliquer ce que sont :

    la loi des sinus
    ...
    Soit un triangle ABC. On note a=BC, b=AC, c=AB, A l'angle en A, B l'angle en B, C l'angle en C.
    alors a/sin(A)=b/sin(B)=c/sin(C0
    Citation Envoyé par shokin
    l'inégalité arithmético-géométrique
    Soient x1,...xn des reels positifs.
    Alors (x1+...+xn)/n>=(x1.x2....xn)^(1/n)

    Citation Envoyé par shokin
    la concavité de sin sur [0,pi]
    Soit f une fonction concave sur un intervalle I. (f'' est negative sur I)
    Soient x1,..xn des reels appartenant a I.
    Soient i1,...,in des reels dont la somme est 1.

    Alors:
    f(i1.x1+...+in.xn)>=i1.f(x1)+. ..+in.f(xn)

    Citation Envoyé par shokin
    l'inégalité d'Erdos Mordell.
    Soit A1,..,An un polygone convexe. Soit P un point a l'interieur de celui-ci.
    Soit Bi le projete de P sur le cote AiA(i+1), avec les indices consideres modulo n.
    Alors:
    PA1+PA2+---+PAn>= 1/cos(pi/n)(PB1+....+PBn).

    Dans un triangle ABC:
    Soit P un point a l'interieur de celui-ci.
    Soit x=PA, y=PB, z=PC ; a=BC, b=AC, c=AB
    Alors a+b+c>=2(x+y+z)

  24. #23
    shokin

    Talking Re : Geometrie

    merci Lord,

    en fait les deux premières, lois du sinus et inégalité arithmético-géométrique, je les connaissais, sans en connaître le nom. Quant aux deux suivantes, c'est du nouveau.

    j'ai enfin compris ta démo, encore faut-il que je me mette à exploiter ces 4 théorèmes, pour que certains problème relèvent du domaine trivial...

    je vais, avec connaissance de ces théorèmes, de chercher une démo simple, rapide, efficace, facile à comprendre... à moins que tu ne me démontres qu'il n'en existe pas...
    Pardon, humilité, humour, hasard, tolérance, partage, curiosité et diversité => liberté et sérénité.

  25. #24
    Quinto

    Re : Geometrie

    Citation Envoyé par Lord
    Soit f une fonction concave sur un intervalle I. (f'' est negative sur I)
    Soient x1,..xn des reels appartenant a I.
    Soient i1,...,in des reels dont la somme est 1.

    Alors:
    f(i1.x1+...+in.xn)>=i1.f(x1)+. ..+in.f(xn)
    C'est pas ca la concavité....
    La fonction valeur absolue est convexe et pas dérivable, alors encore moins 2fois dérivable...

    La concavité c'est ce que tu donnes en dernier, et on a pas besoin d'autres informations que la continuité de f.

    En fait, une fonction est concave si la portion du plan au dessus de ta courbe l'est...

Discussions similaires

  1. géométrie
    Par invite572ebd1a dans le forum Mathématiques du supérieur
    Réponses: 17
    Dernier message: 30/11/2007, 18h35
  2. [1°S] Géométrie
    Par invite2220c077 dans le forum Mathématiques du collège et du lycée
    Réponses: 2
    Dernier message: 28/11/2007, 07h55
  3. Geométrie
    Par invite2f416c20 dans le forum Mathématiques du collège et du lycée
    Réponses: 2
    Dernier message: 23/11/2006, 15h38
  4. Pb De Geometrie Tv
    Par invite2c097e4b dans le forum Dépannage
    Réponses: 4
    Dernier message: 26/07/2006, 19h17
  5. géométrie
    Par invite9079ea52 dans le forum Mathématiques du supérieur
    Réponses: 0
    Dernier message: 24/09/2004, 20h52